LSAT and Law School Admissions Forum

Get expert LSAT preparation and law school admissions advice from PowerScore Test Preparation.

 Administrator
PowerScore Staff
  • PowerScore Staff
  • Posts: 8917
  • Joined: Feb 02, 2011
|
#98254
Complete Question Explanation

The correct answer choice is (D).

Answer choice (A):

Answer choice (B):

Answer choice (C):

Answer choice (D): This is the correct answer choice.

Answer choice (E):

This explanation is still in progress. Please post any questions below!
 quan-tang@hotmail.com
  • Posts: 35
  • Joined: Sep 18, 2022
|
#98833
Hi, is there a quicker way to do this except trial and error?
User avatar
 Paul Popa
PowerScore Staff
  • PowerScore Staff
  • Posts: 64
  • Joined: Sep 20, 2022
|
#98850
Hi Quan,

That is a great question. I'd be happy to see other PowerScore staff comment with their thoughts, but there are unfortunately questions like this one where trial and error is the quickest way to solve them (unless work on a different question allows you to knock off an answer choice or two). What I would recommend on logic game questions that require to try out each answer choice is to start at answer choice E and work your way up. I've noticed that LSAC loves to make D or E the correct answer to these types of questions, as we see here. Hope this helps!
 Adam Tyson
PowerScore Staff
  • PowerScore Staff
  • Posts: 5153
  • Joined: Apr 14, 2011
|
#98854
I'll second Paul's view on this frustrating question, but I'll add that you could also use the work you've done in the original diagram to help narrow things down, IF you did a little more work. For example, when I attacked the setup to this game, I tried a few hypotheticals. I based three of those templates off of the combination of the first rule and the last rule:

1. What if I have G, F, and S? V and I must be out. That leaves H, Q, and R, but I cannot include both H and Q, so R is in. I therefore get GFSR and H/Q.

2. What if I have G, F, and V? Then Q and R must be in, with H, I, and S out. GFVQR is the solution.

3. What if I have H and F? Q, V, and I are out, so everyone else must be in. HFSGR is allowed.

In all of these cases, I is out, which is interesting. Makes me think about what would happen if I was in, and that got me the same scenarios that you may have seen in our setup explanation for this game.

How do I use this to attack this question? I use those three diagrams to eliminate answer choices.

A: Looks like this could happen in either scenario that started with G, so this is out.
B: This happens in one of my G situations, so also out.
C: This also happens in one of my G diagrams, so it's a loser.
D: I haven't seen a situation where this happens yet, so I will keep it as a contender.
E: All three of my scenarios allowed this to happen, so it's out, and D is, by process of elimination, the correct answer, with no need to do any testing at all! The work at the beginning paves the way for fast, easy answer selection later in the game.

This is my general philosophy in Logic Games: more work at the beginning means less work later on.

Oh, and those three templates led me to do one more. Since I already tried every possible scenario that involved H or G, what if they were both out? That means I would be left to choose from these 6 variables:

FIQRSV

But the last rule makes it so that I can never have all three of F, S, and V, so one of those has to be out, and the other three must be in! That would look like this:

IQR (F,S,V)

Four templates, no fuss!
User avatar
 iluvscarbs
  • Posts: 1
  • Joined: Aug 30, 2023
|
#102977
Could someone let me know if my thought process is correct?

I used the contrapositive of rule 1 (if F is not selected, then G AND H are not selected). Since we know I is out, that means only 2 more paintings can also be out. However, if we put F as out, then G and H would also have to be out, but that's now 4 paintings out instead of 3. Meaning F HAS to be in.

What happens if F is in? Then exactly one of S or V is selected. Meaning S and V cannot both be chosen, which makes answer choice D correct.
 Rachael Wilkenfeld
PowerScore Staff
  • PowerScore Staff
  • Posts: 1358
  • Joined: Dec 15, 2011
|
#102988
That's good reasoning there, iluvscarbs. I went at it very similarly. I thought about who can't be selected together. I have a lot of rules where H doesn't go with someone, but H isn't in any of these answer choices. I have my GS rule, but that just tells me the same thing the question stem does---I is out. The only other rule that forces someone out is the F--->SV rule. If I is out, and S and V are both out, we ALSO couldn't have F. That's four out, and we only have three out-slots.

In sum, for me, I looked to the rules which forced someone out, because I being out really shrinks that out group.

Great work!

Get the most out of your LSAT Prep Plus subscription.

Analyze and track your performance with our Testing and Analytics Package.